Calculating Fourier series of $cos^2(t)$ gives unexpected resultUse WolframAlpha to compute the real Fourier series of a functionIs it possible for cosine functions to have Fourier sine series expressions or sine functions to have Fourier cosine series expressions?Finding Fourier series of $sin^2 x$ (STILL not clear - read comments)The coefficients of the Fourier series of the product of two real valued functionsFourier Series; odd and even half-range expansionAfter calculating Fourier series coefficients for $x(t)=2 cos(4t) + 4 sin(10t)$, why am I getting all zeroes for all coefficients?Equivalence of two definitions of Fourier Series(Trigonometric) Fourier series of sawtooth integralEven and odd functions with Fourier seriesFourier series of a fourier series.

Why do phishing e-mails use faked e-mail addresses instead of the real one?

After `ssh` without `-X` to a machine, is it possible to change `$DISPLAY` to make it work like `ssh -X`?

Does "Until when" sound natural for native speakers?

Is it a Cyclops number? "Nobody" knows!

Is it possible to avoid unpacking when merging Association?

Was it really inappropriate to write a pull request for the company I interviewed with?

Do I really need to have a scientific explanation for my premise?

Outlet with 3 sets of wires

Are small insurances worth it?

How does Ehrenfest's theorem apply to the quantum harmonic oscillator?

What will happen if my luggage gets delayed?

In the late 1940’s to early 1950’s what technology was available that could melt a LOT of ice?

Haman going to the second feast dirty

Why does cron require MTA for logging?

Street obstacles in New Zealand

Proving a statement about real numbers

What is Tony Stark injecting into himself in Iron Man 3?

Is it possible to find 2014 distinct positive integers whose sum is divisible by each of them?

How to write a chaotic neutral protagonist and prevent my readers from thinking they are evil?

What is the generally accepted pronunciation of “topoi”?

Getting the || sign while using Kurier

School performs periodic password audits. Is my password compromised?

Can one live in the U.S. and not use a credit card?

How do we create new idioms and use them in a novel?



Calculating Fourier series of $cos^2(t)$ gives unexpected result


Use WolframAlpha to compute the real Fourier series of a functionIs it possible for cosine functions to have Fourier sine series expressions or sine functions to have Fourier cosine series expressions?Finding Fourier series of $sin^2 x$ (STILL not clear - read comments)The coefficients of the Fourier series of the product of two real valued functionsFourier Series; odd and even half-range expansionAfter calculating Fourier series coefficients for $x(t)=2 cos(4t) + 4 sin(10t)$, why am I getting all zeroes for all coefficients?Equivalence of two definitions of Fourier Series(Trigonometric) Fourier series of sawtooth integralEven and odd functions with Fourier seriesFourier series of a fourier series.













0












$begingroup$


As I understand it:



$cos^2(t)$ is even because it is a product of two even functions $cos(t)$.



The Fourier series and Fourier cosine series of an even function is the same link.



So in the fourier series expansion $cos^2(t)=a_0 over 2+sum_n=1^infty(a_ncos(n omega t) + b_n sin(n omega t))$, I expect $a_nneq 0$ and $b_n=0$.



I try to get the the coefficients with wolfram alpha like this:



$a_n$ with FourierCosCoefficient[$cos(t)^2,t,n$] gives zero link



$b_n$ with FourierSinCoefficient[$cos(t)^2,t,n$] gives non-zero link



Which of my assumptions are wrong?



(I also got $a_n=0$ when calculating by hand so the question is not primarily about wolfram alpha but about the relation between Fourier series, Fourier cosine series, and even functions)










share|cite|improve this question









New contributor




aardvark is a new contributor to this site. Take care in asking for clarification, commenting, and answering.
Check out our Code of Conduct.







$endgroup$











  • $begingroup$
    Looks to me that already $a_0 = frac12 ne 0$... Can you detail your computations ?
    $endgroup$
    – Gâteau-Gallois
    yesterday






  • 1




    $begingroup$
    I'm not familiar enough with WolframAlpha to say, but it seems to me this is likely to be a misunderstanding of what the function does. For example, where's the $omega$ in that call? Maybe it's trying to project the function on a different period? As it stands, $cos^2(t) = 1/2+1/2cos(2t)$, so that's all your Fourier series.
    $endgroup$
    – Okarin
    yesterday










  • $begingroup$
    The correct answer is $a_0=a_2=frac 1 2$ and all other coefficients $0$.
    $endgroup$
    – Kavi Rama Murthy
    yesterday










  • $begingroup$
    Surprisingly Mathematica gives the same answer. OTOH to the integral FourierCosCoefficient[(1+Cos[2t])/2,t,n]it gives the expected (DiscreteDelta[n-2]+2 DiscreteDelta[n])/2
    $endgroup$
    – Jyrki Lahtonen
    yesterday







  • 1




    $begingroup$
    FWIW, I asked this at Mathematica.SE.
    $endgroup$
    – Jyrki Lahtonen
    yesterday















0












$begingroup$


As I understand it:



$cos^2(t)$ is even because it is a product of two even functions $cos(t)$.



The Fourier series and Fourier cosine series of an even function is the same link.



So in the fourier series expansion $cos^2(t)=a_0 over 2+sum_n=1^infty(a_ncos(n omega t) + b_n sin(n omega t))$, I expect $a_nneq 0$ and $b_n=0$.



I try to get the the coefficients with wolfram alpha like this:



$a_n$ with FourierCosCoefficient[$cos(t)^2,t,n$] gives zero link



$b_n$ with FourierSinCoefficient[$cos(t)^2,t,n$] gives non-zero link



Which of my assumptions are wrong?



(I also got $a_n=0$ when calculating by hand so the question is not primarily about wolfram alpha but about the relation between Fourier series, Fourier cosine series, and even functions)










share|cite|improve this question









New contributor




aardvark is a new contributor to this site. Take care in asking for clarification, commenting, and answering.
Check out our Code of Conduct.







$endgroup$











  • $begingroup$
    Looks to me that already $a_0 = frac12 ne 0$... Can you detail your computations ?
    $endgroup$
    – Gâteau-Gallois
    yesterday






  • 1




    $begingroup$
    I'm not familiar enough with WolframAlpha to say, but it seems to me this is likely to be a misunderstanding of what the function does. For example, where's the $omega$ in that call? Maybe it's trying to project the function on a different period? As it stands, $cos^2(t) = 1/2+1/2cos(2t)$, so that's all your Fourier series.
    $endgroup$
    – Okarin
    yesterday










  • $begingroup$
    The correct answer is $a_0=a_2=frac 1 2$ and all other coefficients $0$.
    $endgroup$
    – Kavi Rama Murthy
    yesterday










  • $begingroup$
    Surprisingly Mathematica gives the same answer. OTOH to the integral FourierCosCoefficient[(1+Cos[2t])/2,t,n]it gives the expected (DiscreteDelta[n-2]+2 DiscreteDelta[n])/2
    $endgroup$
    – Jyrki Lahtonen
    yesterday







  • 1




    $begingroup$
    FWIW, I asked this at Mathematica.SE.
    $endgroup$
    – Jyrki Lahtonen
    yesterday













0












0








0





$begingroup$


As I understand it:



$cos^2(t)$ is even because it is a product of two even functions $cos(t)$.



The Fourier series and Fourier cosine series of an even function is the same link.



So in the fourier series expansion $cos^2(t)=a_0 over 2+sum_n=1^infty(a_ncos(n omega t) + b_n sin(n omega t))$, I expect $a_nneq 0$ and $b_n=0$.



I try to get the the coefficients with wolfram alpha like this:



$a_n$ with FourierCosCoefficient[$cos(t)^2,t,n$] gives zero link



$b_n$ with FourierSinCoefficient[$cos(t)^2,t,n$] gives non-zero link



Which of my assumptions are wrong?



(I also got $a_n=0$ when calculating by hand so the question is not primarily about wolfram alpha but about the relation between Fourier series, Fourier cosine series, and even functions)










share|cite|improve this question









New contributor




aardvark is a new contributor to this site. Take care in asking for clarification, commenting, and answering.
Check out our Code of Conduct.







$endgroup$




As I understand it:



$cos^2(t)$ is even because it is a product of two even functions $cos(t)$.



The Fourier series and Fourier cosine series of an even function is the same link.



So in the fourier series expansion $cos^2(t)=a_0 over 2+sum_n=1^infty(a_ncos(n omega t) + b_n sin(n omega t))$, I expect $a_nneq 0$ and $b_n=0$.



I try to get the the coefficients with wolfram alpha like this:



$a_n$ with FourierCosCoefficient[$cos(t)^2,t,n$] gives zero link



$b_n$ with FourierSinCoefficient[$cos(t)^2,t,n$] gives non-zero link



Which of my assumptions are wrong?



(I also got $a_n=0$ when calculating by hand so the question is not primarily about wolfram alpha but about the relation between Fourier series, Fourier cosine series, and even functions)







fourier-series wolfram-alpha






share|cite|improve this question









New contributor




aardvark is a new contributor to this site. Take care in asking for clarification, commenting, and answering.
Check out our Code of Conduct.











share|cite|improve this question









New contributor




aardvark is a new contributor to this site. Take care in asking for clarification, commenting, and answering.
Check out our Code of Conduct.









share|cite|improve this question




share|cite|improve this question








edited yesterday









Bernard

122k741116




122k741116






New contributor




aardvark is a new contributor to this site. Take care in asking for clarification, commenting, and answering.
Check out our Code of Conduct.









asked yesterday









aardvarkaardvark

1




1




New contributor




aardvark is a new contributor to this site. Take care in asking for clarification, commenting, and answering.
Check out our Code of Conduct.





New contributor





aardvark is a new contributor to this site. Take care in asking for clarification, commenting, and answering.
Check out our Code of Conduct.






aardvark is a new contributor to this site. Take care in asking for clarification, commenting, and answering.
Check out our Code of Conduct.











  • $begingroup$
    Looks to me that already $a_0 = frac12 ne 0$... Can you detail your computations ?
    $endgroup$
    – Gâteau-Gallois
    yesterday






  • 1




    $begingroup$
    I'm not familiar enough with WolframAlpha to say, but it seems to me this is likely to be a misunderstanding of what the function does. For example, where's the $omega$ in that call? Maybe it's trying to project the function on a different period? As it stands, $cos^2(t) = 1/2+1/2cos(2t)$, so that's all your Fourier series.
    $endgroup$
    – Okarin
    yesterday










  • $begingroup$
    The correct answer is $a_0=a_2=frac 1 2$ and all other coefficients $0$.
    $endgroup$
    – Kavi Rama Murthy
    yesterday










  • $begingroup$
    Surprisingly Mathematica gives the same answer. OTOH to the integral FourierCosCoefficient[(1+Cos[2t])/2,t,n]it gives the expected (DiscreteDelta[n-2]+2 DiscreteDelta[n])/2
    $endgroup$
    – Jyrki Lahtonen
    yesterday







  • 1




    $begingroup$
    FWIW, I asked this at Mathematica.SE.
    $endgroup$
    – Jyrki Lahtonen
    yesterday
















  • $begingroup$
    Looks to me that already $a_0 = frac12 ne 0$... Can you detail your computations ?
    $endgroup$
    – Gâteau-Gallois
    yesterday






  • 1




    $begingroup$
    I'm not familiar enough with WolframAlpha to say, but it seems to me this is likely to be a misunderstanding of what the function does. For example, where's the $omega$ in that call? Maybe it's trying to project the function on a different period? As it stands, $cos^2(t) = 1/2+1/2cos(2t)$, so that's all your Fourier series.
    $endgroup$
    – Okarin
    yesterday










  • $begingroup$
    The correct answer is $a_0=a_2=frac 1 2$ and all other coefficients $0$.
    $endgroup$
    – Kavi Rama Murthy
    yesterday










  • $begingroup$
    Surprisingly Mathematica gives the same answer. OTOH to the integral FourierCosCoefficient[(1+Cos[2t])/2,t,n]it gives the expected (DiscreteDelta[n-2]+2 DiscreteDelta[n])/2
    $endgroup$
    – Jyrki Lahtonen
    yesterday







  • 1




    $begingroup$
    FWIW, I asked this at Mathematica.SE.
    $endgroup$
    – Jyrki Lahtonen
    yesterday















$begingroup$
Looks to me that already $a_0 = frac12 ne 0$... Can you detail your computations ?
$endgroup$
– Gâteau-Gallois
yesterday




$begingroup$
Looks to me that already $a_0 = frac12 ne 0$... Can you detail your computations ?
$endgroup$
– Gâteau-Gallois
yesterday




1




1




$begingroup$
I'm not familiar enough with WolframAlpha to say, but it seems to me this is likely to be a misunderstanding of what the function does. For example, where's the $omega$ in that call? Maybe it's trying to project the function on a different period? As it stands, $cos^2(t) = 1/2+1/2cos(2t)$, so that's all your Fourier series.
$endgroup$
– Okarin
yesterday




$begingroup$
I'm not familiar enough with WolframAlpha to say, but it seems to me this is likely to be a misunderstanding of what the function does. For example, where's the $omega$ in that call? Maybe it's trying to project the function on a different period? As it stands, $cos^2(t) = 1/2+1/2cos(2t)$, so that's all your Fourier series.
$endgroup$
– Okarin
yesterday












$begingroup$
The correct answer is $a_0=a_2=frac 1 2$ and all other coefficients $0$.
$endgroup$
– Kavi Rama Murthy
yesterday




$begingroup$
The correct answer is $a_0=a_2=frac 1 2$ and all other coefficients $0$.
$endgroup$
– Kavi Rama Murthy
yesterday












$begingroup$
Surprisingly Mathematica gives the same answer. OTOH to the integral FourierCosCoefficient[(1+Cos[2t])/2,t,n]it gives the expected (DiscreteDelta[n-2]+2 DiscreteDelta[n])/2
$endgroup$
– Jyrki Lahtonen
yesterday





$begingroup$
Surprisingly Mathematica gives the same answer. OTOH to the integral FourierCosCoefficient[(1+Cos[2t])/2,t,n]it gives the expected (DiscreteDelta[n-2]+2 DiscreteDelta[n])/2
$endgroup$
– Jyrki Lahtonen
yesterday





1




1




$begingroup$
FWIW, I asked this at Mathematica.SE.
$endgroup$
– Jyrki Lahtonen
yesterday




$begingroup$
FWIW, I asked this at Mathematica.SE.
$endgroup$
– Jyrki Lahtonen
yesterday










3 Answers
3






active

oldest

votes


















3












$begingroup$

Just read the documentation:
https://reference.wolfram.com/language/ref/FourierSinCoefficient.html



The command FourierSinCoefficient does not compute the coefficients $b_n$ in the full Fourier series, but the coefficients in the Fourier sine series (= the full Fourier series of the odd extension of the function in question).






share|cite|improve this answer









$endgroup$












  • $begingroup$
    Thanks, this shows why I was wrong to assume I could calculate $b_n$ with FourierSinCoefficient. I still think I should be able to calculate $a_n$ with FourierCosCoefficient.
    $endgroup$
    – aardvark
    yesterday


















1












$begingroup$

The simplest way to find the Fourier series of this function is to write it as $frac 1+cos(2t) 2$. This expression is in fact the Fourier series!.






share|cite|improve this answer









$endgroup$








  • 2




    $begingroup$
    I'm sure we all know this. The question is why WolframAlpha gives a different answer.
    $endgroup$
    – Jyrki Lahtonen
    yesterday


















0












$begingroup$

It is clear that, for each $ninmathbb N$,$$int_-pi^picos^2(t)sin(nt),mathrm dt=0,$$since $tmapstocos^2(t)sin(nt)$ is an odd function.



On the other hand,$$a_0=frac1piint_-pi^picos^2(t),mathrm dt=1neq0.$$It turns out that $a_1=0$, but $a_2=frac12neq0$.






share|cite|improve this answer









$endgroup$












  • $begingroup$
    I'm sure we all know this. The question is why WolframAlpha gives a different answer.
    $endgroup$
    – Jyrki Lahtonen
    yesterday






  • 1




    $begingroup$
    No, we don't all know this, since the OP claimed to have obtained $a_n=0$ while calculating by hand.
    $endgroup$
    – José Carlos Santos
    yesterday










Your Answer





StackExchange.ifUsing("editor", function ()
return StackExchange.using("mathjaxEditing", function ()
StackExchange.MarkdownEditor.creationCallbacks.add(function (editor, postfix)
StackExchange.mathjaxEditing.prepareWmdForMathJax(editor, postfix, [["$", "$"], ["\\(","\\)"]]);
);
);
, "mathjax-editing");

StackExchange.ready(function()
var channelOptions =
tags: "".split(" "),
id: "69"
;
initTagRenderer("".split(" "), "".split(" "), channelOptions);

StackExchange.using("externalEditor", function()
// Have to fire editor after snippets, if snippets enabled
if (StackExchange.settings.snippets.snippetsEnabled)
StackExchange.using("snippets", function()
createEditor();
);

else
createEditor();

);

function createEditor()
StackExchange.prepareEditor(
heartbeatType: 'answer',
autoActivateHeartbeat: false,
convertImagesToLinks: true,
noModals: true,
showLowRepImageUploadWarning: true,
reputationToPostImages: 10,
bindNavPrevention: true,
postfix: "",
imageUploader:
brandingHtml: "Powered by u003ca class="icon-imgur-white" href="https://imgur.com/"u003eu003c/au003e",
contentPolicyHtml: "User contributions licensed under u003ca href="https://creativecommons.org/licenses/by-sa/3.0/"u003ecc by-sa 3.0 with attribution requiredu003c/au003e u003ca href="https://stackoverflow.com/legal/content-policy"u003e(content policy)u003c/au003e",
allowUrls: true
,
noCode: true, onDemand: true,
discardSelector: ".discard-answer"
,immediatelyShowMarkdownHelp:true
);



);






aardvark is a new contributor. Be nice, and check out our Code of Conduct.









draft saved

draft discarded


















StackExchange.ready(
function ()
StackExchange.openid.initPostLogin('.new-post-login', 'https%3a%2f%2fmath.stackexchange.com%2fquestions%2f3141024%2fcalculating-fourier-series-of-cos2t-gives-unexpected-result%23new-answer', 'question_page');

);

Post as a guest















Required, but never shown

























3 Answers
3






active

oldest

votes








3 Answers
3






active

oldest

votes









active

oldest

votes






active

oldest

votes









3












$begingroup$

Just read the documentation:
https://reference.wolfram.com/language/ref/FourierSinCoefficient.html



The command FourierSinCoefficient does not compute the coefficients $b_n$ in the full Fourier series, but the coefficients in the Fourier sine series (= the full Fourier series of the odd extension of the function in question).






share|cite|improve this answer









$endgroup$












  • $begingroup$
    Thanks, this shows why I was wrong to assume I could calculate $b_n$ with FourierSinCoefficient. I still think I should be able to calculate $a_n$ with FourierCosCoefficient.
    $endgroup$
    – aardvark
    yesterday















3












$begingroup$

Just read the documentation:
https://reference.wolfram.com/language/ref/FourierSinCoefficient.html



The command FourierSinCoefficient does not compute the coefficients $b_n$ in the full Fourier series, but the coefficients in the Fourier sine series (= the full Fourier series of the odd extension of the function in question).






share|cite|improve this answer









$endgroup$












  • $begingroup$
    Thanks, this shows why I was wrong to assume I could calculate $b_n$ with FourierSinCoefficient. I still think I should be able to calculate $a_n$ with FourierCosCoefficient.
    $endgroup$
    – aardvark
    yesterday













3












3








3





$begingroup$

Just read the documentation:
https://reference.wolfram.com/language/ref/FourierSinCoefficient.html



The command FourierSinCoefficient does not compute the coefficients $b_n$ in the full Fourier series, but the coefficients in the Fourier sine series (= the full Fourier series of the odd extension of the function in question).






share|cite|improve this answer









$endgroup$



Just read the documentation:
https://reference.wolfram.com/language/ref/FourierSinCoefficient.html



The command FourierSinCoefficient does not compute the coefficients $b_n$ in the full Fourier series, but the coefficients in the Fourier sine series (= the full Fourier series of the odd extension of the function in question).







share|cite|improve this answer












share|cite|improve this answer



share|cite|improve this answer










answered yesterday









Hans LundmarkHans Lundmark

35.8k564115




35.8k564115











  • $begingroup$
    Thanks, this shows why I was wrong to assume I could calculate $b_n$ with FourierSinCoefficient. I still think I should be able to calculate $a_n$ with FourierCosCoefficient.
    $endgroup$
    – aardvark
    yesterday
















  • $begingroup$
    Thanks, this shows why I was wrong to assume I could calculate $b_n$ with FourierSinCoefficient. I still think I should be able to calculate $a_n$ with FourierCosCoefficient.
    $endgroup$
    – aardvark
    yesterday















$begingroup$
Thanks, this shows why I was wrong to assume I could calculate $b_n$ with FourierSinCoefficient. I still think I should be able to calculate $a_n$ with FourierCosCoefficient.
$endgroup$
– aardvark
yesterday




$begingroup$
Thanks, this shows why I was wrong to assume I could calculate $b_n$ with FourierSinCoefficient. I still think I should be able to calculate $a_n$ with FourierCosCoefficient.
$endgroup$
– aardvark
yesterday











1












$begingroup$

The simplest way to find the Fourier series of this function is to write it as $frac 1+cos(2t) 2$. This expression is in fact the Fourier series!.






share|cite|improve this answer









$endgroup$








  • 2




    $begingroup$
    I'm sure we all know this. The question is why WolframAlpha gives a different answer.
    $endgroup$
    – Jyrki Lahtonen
    yesterday















1












$begingroup$

The simplest way to find the Fourier series of this function is to write it as $frac 1+cos(2t) 2$. This expression is in fact the Fourier series!.






share|cite|improve this answer









$endgroup$








  • 2




    $begingroup$
    I'm sure we all know this. The question is why WolframAlpha gives a different answer.
    $endgroup$
    – Jyrki Lahtonen
    yesterday













1












1








1





$begingroup$

The simplest way to find the Fourier series of this function is to write it as $frac 1+cos(2t) 2$. This expression is in fact the Fourier series!.






share|cite|improve this answer









$endgroup$



The simplest way to find the Fourier series of this function is to write it as $frac 1+cos(2t) 2$. This expression is in fact the Fourier series!.







share|cite|improve this answer












share|cite|improve this answer



share|cite|improve this answer










answered yesterday









Kavi Rama MurthyKavi Rama Murthy

66k42867




66k42867







  • 2




    $begingroup$
    I'm sure we all know this. The question is why WolframAlpha gives a different answer.
    $endgroup$
    – Jyrki Lahtonen
    yesterday












  • 2




    $begingroup$
    I'm sure we all know this. The question is why WolframAlpha gives a different answer.
    $endgroup$
    – Jyrki Lahtonen
    yesterday







2




2




$begingroup$
I'm sure we all know this. The question is why WolframAlpha gives a different answer.
$endgroup$
– Jyrki Lahtonen
yesterday




$begingroup$
I'm sure we all know this. The question is why WolframAlpha gives a different answer.
$endgroup$
– Jyrki Lahtonen
yesterday











0












$begingroup$

It is clear that, for each $ninmathbb N$,$$int_-pi^picos^2(t)sin(nt),mathrm dt=0,$$since $tmapstocos^2(t)sin(nt)$ is an odd function.



On the other hand,$$a_0=frac1piint_-pi^picos^2(t),mathrm dt=1neq0.$$It turns out that $a_1=0$, but $a_2=frac12neq0$.






share|cite|improve this answer









$endgroup$












  • $begingroup$
    I'm sure we all know this. The question is why WolframAlpha gives a different answer.
    $endgroup$
    – Jyrki Lahtonen
    yesterday






  • 1




    $begingroup$
    No, we don't all know this, since the OP claimed to have obtained $a_n=0$ while calculating by hand.
    $endgroup$
    – José Carlos Santos
    yesterday















0












$begingroup$

It is clear that, for each $ninmathbb N$,$$int_-pi^picos^2(t)sin(nt),mathrm dt=0,$$since $tmapstocos^2(t)sin(nt)$ is an odd function.



On the other hand,$$a_0=frac1piint_-pi^picos^2(t),mathrm dt=1neq0.$$It turns out that $a_1=0$, but $a_2=frac12neq0$.






share|cite|improve this answer









$endgroup$












  • $begingroup$
    I'm sure we all know this. The question is why WolframAlpha gives a different answer.
    $endgroup$
    – Jyrki Lahtonen
    yesterday






  • 1




    $begingroup$
    No, we don't all know this, since the OP claimed to have obtained $a_n=0$ while calculating by hand.
    $endgroup$
    – José Carlos Santos
    yesterday













0












0








0





$begingroup$

It is clear that, for each $ninmathbb N$,$$int_-pi^picos^2(t)sin(nt),mathrm dt=0,$$since $tmapstocos^2(t)sin(nt)$ is an odd function.



On the other hand,$$a_0=frac1piint_-pi^picos^2(t),mathrm dt=1neq0.$$It turns out that $a_1=0$, but $a_2=frac12neq0$.






share|cite|improve this answer









$endgroup$



It is clear that, for each $ninmathbb N$,$$int_-pi^picos^2(t)sin(nt),mathrm dt=0,$$since $tmapstocos^2(t)sin(nt)$ is an odd function.



On the other hand,$$a_0=frac1piint_-pi^picos^2(t),mathrm dt=1neq0.$$It turns out that $a_1=0$, but $a_2=frac12neq0$.







share|cite|improve this answer












share|cite|improve this answer



share|cite|improve this answer










answered yesterday









José Carlos SantosJosé Carlos Santos

166k22132235




166k22132235











  • $begingroup$
    I'm sure we all know this. The question is why WolframAlpha gives a different answer.
    $endgroup$
    – Jyrki Lahtonen
    yesterday






  • 1




    $begingroup$
    No, we don't all know this, since the OP claimed to have obtained $a_n=0$ while calculating by hand.
    $endgroup$
    – José Carlos Santos
    yesterday
















  • $begingroup$
    I'm sure we all know this. The question is why WolframAlpha gives a different answer.
    $endgroup$
    – Jyrki Lahtonen
    yesterday






  • 1




    $begingroup$
    No, we don't all know this, since the OP claimed to have obtained $a_n=0$ while calculating by hand.
    $endgroup$
    – José Carlos Santos
    yesterday















$begingroup$
I'm sure we all know this. The question is why WolframAlpha gives a different answer.
$endgroup$
– Jyrki Lahtonen
yesterday




$begingroup$
I'm sure we all know this. The question is why WolframAlpha gives a different answer.
$endgroup$
– Jyrki Lahtonen
yesterday




1




1




$begingroup$
No, we don't all know this, since the OP claimed to have obtained $a_n=0$ while calculating by hand.
$endgroup$
– José Carlos Santos
yesterday




$begingroup$
No, we don't all know this, since the OP claimed to have obtained $a_n=0$ while calculating by hand.
$endgroup$
– José Carlos Santos
yesterday










aardvark is a new contributor. Be nice, and check out our Code of Conduct.









draft saved

draft discarded


















aardvark is a new contributor. Be nice, and check out our Code of Conduct.












aardvark is a new contributor. Be nice, and check out our Code of Conduct.











aardvark is a new contributor. Be nice, and check out our Code of Conduct.














Thanks for contributing an answer to Mathematics Stack Exchange!


  • Please be sure to answer the question. Provide details and share your research!

But avoid


  • Asking for help, clarification, or responding to other answers.

  • Making statements based on opinion; back them up with references or personal experience.

Use MathJax to format equations. MathJax reference.


To learn more, see our tips on writing great answers.




draft saved


draft discarded














StackExchange.ready(
function ()
StackExchange.openid.initPostLogin('.new-post-login', 'https%3a%2f%2fmath.stackexchange.com%2fquestions%2f3141024%2fcalculating-fourier-series-of-cos2t-gives-unexpected-result%23new-answer', 'question_page');

);

Post as a guest















Required, but never shown





















































Required, but never shown














Required, but never shown












Required, but never shown







Required, but never shown

































Required, but never shown














Required, but never shown












Required, but never shown







Required, but never shown







Popular posts from this blog

Lowndes Grove History Architecture References Navigation menu32°48′6″N 79°57′58″W / 32.80167°N 79.96611°W / 32.80167; -79.9661132°48′6″N 79°57′58″W / 32.80167°N 79.96611°W / 32.80167; -79.9661178002500"National Register Information System"Historic houses of South Carolina"Lowndes Grove""+32° 48' 6.00", −79° 57' 58.00""Lowndes Grove, Charleston County (260 St. Margaret St., Charleston)""Lowndes Grove"The Charleston ExpositionIt Happened in South Carolina"Lowndes Grove (House), Saint Margaret Street & Sixth Avenue, Charleston, Charleston County, SC(Photographs)"Plantations of the Carolina Low Countrye

random experiment with two different functions on unit interval Announcing the arrival of Valued Associate #679: Cesar Manara Planned maintenance scheduled April 23, 2019 at 00:00UTC (8:00pm US/Eastern)Random variable and probability space notionsRandom Walk with EdgesFinding functions where the increase over a random interval is Poisson distributedNumber of days until dayCan an observed event in fact be of zero probability?Unit random processmodels of coins and uniform distributionHow to get the number of successes given $n$ trials , probability $P$ and a random variable $X$Absorbing Markov chain in a computer. Is “almost every” turned into always convergence in computer executions?Stopped random walk is not uniformly integrable

How should I support this large drywall patch? Planned maintenance scheduled April 23, 2019 at 00:00UTC (8:00pm US/Eastern) Announcing the arrival of Valued Associate #679: Cesar Manara Unicorn Meta Zoo #1: Why another podcast?How do I cover large gaps in drywall?How do I keep drywall around a patch from crumbling?Can I glue a second layer of drywall?How to patch long strip on drywall?Large drywall patch: how to avoid bulging seams?Drywall Mesh Patch vs. Bulge? To remove or not to remove?How to fix this drywall job?Prep drywall before backsplashWhat's the best way to fix this horrible drywall patch job?Drywall patching using 3M Patch Plus Primer